Topology Help: Proving Open Sets in T[SUB]C for X and C Collection"

  • Thread starter Thread starter son
  • Start date Start date
  • Tags Tags
    Topology
Click For Summary
In the discussion, the focus is on proving that every set in the collection C is an open set within the topology T_C generated by the basis β_C. It is established that each element of C can be expressed as a finite intersection of sets from C, which means they belong to the basis β_C. Since all elements of β_C are defined as open sets in the topology T_C, it follows that every set in C is open. The proof hinges on recognizing that sets in C are part of the basis and thus satisfy the criteria for openness. This establishes the relationship between the sub-basis C and the topology T_C effectively.
son
Messages
7
Reaction score
0
let X be a set and C be a collection of subsets of X whose union equal X. let βC the collection of all subsets of X that can be expressed as an intersection of finitely many of the sets from C.

let TC be the topology generated by the basis βC.

prove that every set in C is an open set in the topology TC.










Homework Statement


Homework Equations


The Attempt at a Solution

 
Physics news on Phys.org


any ideas/attempts?
 


Essentially what you need to do is show that any subset A of C can be expressed as a union of elements of B_C.

C in this case is a sub-basis of X, and T_c the topology that this subbasis induces, so to speak.
 


this is what i came up with... but this is not consider a proof...

every element in C will be in the basis β_C. Let U be in C then U is the finite intersection of elements in C, for example U = U ∩ U. It follows that U ∈ β_C. And by the definition of the topology, every element in β_C is open, so U is thus open.
 
Question: A clock's minute hand has length 4 and its hour hand has length 3. What is the distance between the tips at the moment when it is increasing most rapidly?(Putnam Exam Question) Answer: Making assumption that both the hands moves at constant angular velocities, the answer is ## \sqrt{7} .## But don't you think this assumption is somewhat doubtful and wrong?

Similar threads

  • · Replies 1 ·
Replies
1
Views
2K
  • · Replies 12 ·
Replies
12
Views
2K
  • · Replies 12 ·
Replies
12
Views
2K
  • · Replies 12 ·
Replies
12
Views
2K
  • · Replies 3 ·
Replies
3
Views
2K
  • · Replies 1 ·
Replies
1
Views
2K
Replies
20
Views
4K
  • · Replies 8 ·
Replies
8
Views
2K
  • · Replies 5 ·
Replies
5
Views
2K
  • · Replies 4 ·
Replies
4
Views
3K